Sie sind auf Seite 1von 37

Rewrite1 Question 61 1.

105231

61. How many sisters does Horace Horton have given that Horace Horton is a man? (1) Horace Horton has as many sisters as brothers (2) Horace Horton s sister Annabelle Horton has twice as many brothers as sisters (A) (B) (C) (D) (E) Statement (1) ALONE is sufficient, but statement (2) alone is not sufficient. Statement (2) ALONE is sufficient, but statement (1) alone is not sufficient. BOTH statements TOGETHER are sufficient, but NEITHER statement ALONE is sufficient. EACH statement ALONE is sufficient. Statements (1) and (2) TOGETHER are NOT sufficient.

Question ID:

System of independent, linear equations


Solution In order to answer this question, you should know that no person is his/her own brother/sister. You are asked to find the number of sisters of Horace Horton. There is no information regarding the number of sisters of Horace Horton in the question stem, so let s look into the statements. Statement (1): In this statement, you are told that Horace Horton has as many brothers as sisters. This statement is clearly not sufficient because Horace Horton could have two sisters and two brothers, or he could have five sisters and five brothers, for examples. As a result, the number of sisters of Horace Horton can not be determined from this statement alone. Statement (1) ALONE is NOT sufficient. Statement (2): In this statement, you are told that Annabelle Horton has twice as many brothers as sisters. This statement alone does not provide sufficient information either, since Annabelle Horace could have four brothers and two sisters, for example, or she could have brothers and seven sisters, for instance. More information is required to answer this question. Statement (2) ALONE is NOT sufficient. BOTH statements TOGETHER: In the first statement, you are told that the number of brothers and sisters of Horace Horton are equal. In the second statement, you are told that the number of brothers of Annabelle Horton is

twice the number of her sisters. If the two statements seem to provide contradictory information, then there is no need to panic. You only need to remember that Horace Horton is not his own brother but Annabelle Horton s brother. Likewise, Annabelle Horton is not her own sister but Horace Horton s sister. Thus, if you let the number of brothers and sisters of Horace Horton be and , respectively, then statement tells you that . As Annabelle Horton has one more brother than Horace Horton (because Horace Horton is Annabelle Horton s brother but not his own brother) and one less sister than Horace Horton (because Annabelle Horton is Horace Horton s sister but not her own sister), statement tells you that . When simplified, equation becomes: . This can be further simplified once you add to both sides of the last equation: . Equations and form a system of two linear, independent equations in two unknowns, and , that can be solved for and thus the question can be answered. BOTH statements TOGETHER are sufficient but NEITHER statement ALONE is sufficient. The correct answer is C. Additional Notes A system of linear equations is said to be independent if none of the equations can be algebraically manipulated to look like the other. In essence, this means none of the equations repeats information already provided by the other equations.

Rewrite Question 62 0.113265

Note: Figure not drawn to scale

62. In the figure above, (

has an area of

. What is the area of (

1. Quadrilateral 2. The length of side (A) (B) (C) (D) (E)

has an area of

. is .

is and the length of side

Statement (1) ALONE is sufficient, but statement (2) alone is not sufficient. Statement (2) ALONE is sufficient, but statement (1) alone is not sufficient. BOTH statements TOGETHER are sufficient, but NEITHER statement ALONE is sufficient. EACH statement ALONE is sufficient. Statements (1) and (2) TOGETHER are NOT sufficient.

Question ID:

Area of a triangle; Area, perimeter


Solution The cautionary note below the figure informs you that the figure itself is not drawn to scale. Hence, you should be very careful of deducing anything from the figure itself. Statement (1): In this statement, you are told that the area of quadrilateral is . The question stem informs you that the area of ( is . As line segment divides the quadrilateral into two triangles, namely ( and ( , the area of quadrilateral equals the sum of the areas of these two triangles.

Note: Figure not drawn to scale

In other words, you may write that:     (  ( . Subtracting  ( from both sides of this equation, you obtain the equation:  (     ( . The question stem provides you with the  ( and this statement tells you that    is . Hence, answering the question is now straightforward.

Statement (1) ALONE is sufficient. Statement (2): This statement informs you that the length of side is and the length of side is . If angle were a right-angle, then you could easily calculate the area of ( by multiplying the lengths of sides and and dividing the product by . However, as you are not told whether the interior angle is a right-angle, you can not determine the answer to the question without additional information. In particular, if you knew the measure of angle , you could calculate the area of ( .

Note: Figure not drawn to scale

Statement (2) ALONE is NOT sufficient. The correct answer is A.

Rewrite Question 63 -2.053415

63. The linear equation contains two unknowns and while is a constant. If a solution to this equation is presented in the form , is a solution to the equation?

1. 2.

is a solution of the equation is a solution of the equation

(A) (B) (C) (D) (E)

Statement (1) ALONE is sufficient, but statement (2) alone is not sufficient. Statement (2) ALONE is sufficient, but statement (1) alone is not sufficient. BOTH statements TOGETHER are sufficient, but NEITHER statement ALONE is sufficient. EACH statement ALONE is sufficient. Statements (1) and (2) TOGETHER are NOT sufficient.

Question ID:

Linear equations
Solution This is one of the trickier questions. Even if you do not make any mistakes in your calculations, you could still end up picking the wrong choice because you need to determine if the question can be answered using the information provided, not whether the answer is affirmative. To determine if any pair of numbers is a solution to the equation , you need to know the value of . In this question, you need to find out if is a solution to equation . By plugging in for and for in equation , you obtain: . Dividing both sides of this equation by produces the equation: . Your task now becomes much simpler. You only need to determine if Statement (1): In this statement, you are told that is a solution of the equation . Plugging in for and for in equation , you obtain the equation: . Hence, this statement essentially tells you that . Thus, and the answer to the question is negative. However, note that the answer can be found using the information in this statement alone and hence this statement is sufficient. Statement (1) ALONE is sufficient. Statement (2): According to this statement, is a solution of the equation . Plugging in for and for into equation , you get: Dividing this equation by , you obtain the equation: . Hence, this statement also tells you that and thus, you know that . This statement answers the question in the negative. However, this statement alone provides enough information to answer the question. Note that you do not have to actually calculate the value of m. As soon as you can see that it can be calculated, you can answer the question. Statement (2) ALONE is sufficient. EACH statement ALONE is sufficient. The correct answer is D. .

Rewrite Question 64 0.025361

64. What does equal?

1. 2.


(A) (B) (C) (D) (E)

Statement (1) ALONE is sufficient, but statement (2) alone is not sufficient. Statement (2) ALONE is sufficient, but statement (1) alone is not sufficient. BOTH statements TOGETHER are sufficient, but NEITHER statement ALONE is sufficient. EACH statement ALONE is sufficient. Statements (1) and (2) TOGETHER are NOT sufficient.

Question ID:

Linear equations; Quadratic equations


Solution Statement (1): You are provided the equation . This is a linear equation in two unknowns which can not be solved to find the value of . Unless you are provided additional information, the value of can not be determined. Statement (1) ALONE is NOT sufficient. Statement (2): In this statement, you are told that . This is a quadratic equation in two unknowns, which cannot be solved for either or . For example, if , then could equal or . As you can see, even if you were provided the value of , there could still be two different values of that would satisfy equation . More information is needed to answer the question. Statement (2) ALONE is NOT sufficient. BOTH statements TOGETHER: From equation , you know that . Adding to both sides of this equation yields: . Squaring both sides of equation produces the equation: . Equating the right sides of equations and , you may write that: .

Subtracting from both sides of equation and multiplying by yields: . Dividing both sides of this equation by produces the equation: . Using either the formula for the solution of a quadratic equation or by factorizing the left side of this expression, you can find that could equal or could equal . If equalled , then from equation , you know that equation, you obtain that . On the other hand, if Subtracting from both sides of this equation would yield: . Adding to both sides of this equalled - , then . .

As two distinct pairs of values of and satisfy both the equations, there is no unique value of that you can find from the two statements combined. BOTH statements TOGETHER are NOT sufficient. The correct answer is E.

Rewrite Question 65 1.123789 ?

65. If and are negative numbers, is

1. The ratio of to is 2.


and the ratio of to is

(A) (B) (C) (D) (E)

Statement (1) ALONE is sufficient, but statement (2) alone is not sufficient. Statement (2) ALONE is sufficient, but statement (1) alone is not sufficient. BOTH statements TOGETHER are sufficient, but NEITHER statement ALONE is sufficient. EACH statement ALONE is sufficient. Statements (1) and (2) TOGETHER are NOT sufficient.

Question ID:

Ratio & Proportion; Linear inequalities


Solution You are told that both and are negative numbers and asked to find if . The stem does not provide much useful information, so let us move to the statements for more information. Statement (1):

Here, you are told that the ratio of itself must be negative.

to

is

and the ratio of and . Since

to

is

. Algebraically, the

previous statement may be represented as

is negative, for to be positive,

To determine , you need to multiply by . and are negative, . For example, if and

. Hence, , then

. Now, as both and since

. Thus, the answer to the question is negative. Statement (1) ALONE is sufficient. Statement (2): According to this statement, . Dividing both sides of this inequality by yields: . Remember that whenever you multiply or divide an inequality by a negative number, the inequality sign flips or reverses . For example, but when you multiply both sides of this inequality by , you get . This statement is sufficient to answer the question. Statement (2) ALONE is sufficient. EACH statement ALONE is sufficient. The correct answer is D. Additional Notes Whenever you multiply or divide an inequality by a negative number, the inequality sign flips or reverses .

Rewrite Question 66 -2.963524 . Is this block of solid a cube?

66. The base of a rectangular block of solid has an area of

1. The area of a side face of the rectangular block is 2. The area of a front face of the rectangular block is

(A) (B) (C) (D)

Statement (1) ALONE is sufficient, but statement (2) alone is not sufficient. Statement (2) ALONE is sufficient, but statement (1) alone is not sufficient. BOTH statements TOGETHER are sufficient, but NEITHER statement ALONE is sufficient. EACH statement ALONE is sufficient.

(E) Statements (1) and (2) TOGETHER are NOT sufficient.

Question ID:

Cube; Additional restrictions, typical tricks


Solution A cube is a three-dimensional rectangular block whose length, width and height are equal to each other. Hence, the question essentially asks if the length , width and height of this rectangular block are equal to each other. The base of this rectangular block has an area of , thus you may write that . This is a single equation in two unknowns, hence you can not determine the values of and . However, if the rectangular block is a cube, then and so and (since neither nor can be negative, only positive values of and are considered). In this case, you would need the height of this rectangular block to also equal for the answer to be affirmative. Let us look into the statements to determine if the height of the rectangular block is . Statement (1): According to this statement, the area of a side face of the rectangular block is . In essence, this tells you that . This is a single equation in two unknowns which can not be solved for either or . However, what you can determine is that the height and length can not each equal . Hence, you know that this rectangular block is not a cube as all three sides of this block are not of the same dimension. This answers the question. Statement (1) ALONE is sufficient. Statement (2): In this statement, you are told that the area of a front face of the rectangular block is . This may be represented algebraically as . As this is a single equation in two unknowns, you can not solve this equation for and , but you can determine that both and can not equal . As a result, the dimensions of the height, width and length of this rectangular block are not the same. Hence, the answer to the question is negative. Statement (2) ALONE is sufficient. EACH statement ALONE is sufficient. The correct answer is D.

Rewrite Question 67 -1.273419

67. If Hillbilly Hick earns five times as much as Billy Bob for every lawn mowed, then how much does Billy Bob earn for every lawn mowed?

1. Hillbilly Hick earns

more than Billy Bob for every lawn mowed.

2. John Joe and Billy Bob together earn half as much as Hillbilly Hick for every lawn mowed by each person.

(A) (B) (C) (D) (E)

Statement (1) ALONE is sufficient, but statement (2) alone is not sufficient. Statement (2) ALONE is sufficient, but statement (1) alone is not sufficient. BOTH statements TOGETHER are sufficient, but NEITHER statement ALONE is sufficient. EACH statement ALONE is sufficient. Statements (1) and (2) TOGETHER are NOT sufficient.

Question ID:

System of independent, linear equations


Solution This is simply an algebraic problem disguised in words. If you let the amount earned by Hillbilly Hick for every lawn mowed be and the amount earned by Billy Bob for every lawn mowed be , then the question stem informs you that . This is a single linear equation in two unknowns which you cannot solve for . If you were provided the value of , or if you could obtain another independent equation in these two unknowns, then you could solve for . Statement (1): According to this statement, Hillbilly Hick earns more than Billy Bob for every lawn mowed. Algebraically, you can represent this information as: . Together with equation , this equation forms a system of independent, linear equations in two unknowns which can be solved to obtain the value of . You do not need to solve for , though, as this is a Data Sufficiency question. If you fail to see how the two equations form a system of independent, linear equations, you should substitute the expression for in equation into equation . Then, you would obtain: which is a linear equation in a single unknown and can be solved for . Statement (1) ALONE is sufficient. Statement (2):

In this statement, you are told that John Joe and Billy Bob together earn half as much as Hillbilly Hick for every lawn mowed by each person. If you let the amount earned by John Joe for every lawn mowed be , then this question informs you that . Together with equation , this equation forms a system of two equations in three unknowns which can not be solved for without an additional independent equation in the three unknowns. If you could form a system of two independent equations in two unknowns or three independent equations in three unknowns, you could solve for . This statement alone is not sufficient to answer the question. Statement (2) ALONE is NOT sufficient. The correct answer is A. Additional Notes A system of equations is independent if none of the equations can be algebraically manipulated to look like any of the remaining equations.

Rewrite Question 68 -2.115487 visited the Prairies last year if there are in total? male and

68. How many residents of female residents in

1. The number of male residents who visited the Prairies last year is one more than the number of female residents who visited the Prairies last year.

2.

of the male residents and of the female residents of year.

visited the Prairies last

(A) (B) (C) (D) (E)

Statement (1) ALONE is sufficient, but statement (2) alone is not sufficient. Statement (2) ALONE is sufficient, but statement (1) alone is not sufficient. BOTH statements TOGETHER are sufficient, but NEITHER statement ALONE is sufficient. EACH statement ALONE is sufficient. Statements (1) and (2) TOGETHER are NOT sufficient.

Question ID:

Linear equations

Solution Statement (1): There are male and female residents of . According to this statement, the number of male visitors to the Prairies is one more than the number of female visitors to the Prairies. However, this does not tell you how many visitors from visited the Prairies last year. For example, it could happen that male visitors and female visitors visited the Prairies last year, bringing the total number of visitors to the Prairies to . On the other hand, it could happen that the number of male visitors was while the number of female visitors was , in which case , the total number of visitors to the Prairies would be . Unless additional information is provided, this question cannot be answered. Statement (1) ALONE is NOT sufficient. Statement (2): According to this statement, the Prairies and of the of the male residents, or male residents visited female residents visited the to the

female residents, or

Prairies. Thus, you can easily calculate that the total number of visitors from Prairies last year equalled . Statement (2) ALONE is sufficient. The correct answer is B.

Rewrite Question 69 1.236597

69. In which year did Billy Bob purchase his first tricycle?

1. When John Joe purchased his first tricycle in his first tricycle.  2. Jim Bob purchased his first tricycle in first tricycle.

, it was

years after Billy Bob purchased

exactly two years before Billy Bob purchased his

(A) Statement (1) ALONE is sufficient, but statement (2) alone is not sufficient.

(B) (C) (D) (E)

Statement (2) ALONE is sufficient, but statement (1) alone is not sufficient. BOTH statements TOGETHER are sufficient, but NEITHER statement ALONE is sufficient. EACH statement ALONE is sufficient. Statements (1) and (2) TOGETHER are NOT sufficient.

Question ID:

Arithmetic operations with fractions; Word Problems


Solution Statement (1): In , John Joe purchased his first tricycle and this was years after Billy Bob purchased his first

tricycle. Because John Joe purchased the tricycle a fractional number of years after Billy Bob, there are two possibilities that arise here. If John Joe purchased his tricycle in November of , for example, or the latter half of then , then , then years prior to that date would be . . On the other , hand, if John Joe purchased his tricycle in March of years prior to that date would be , for instance, or the earlier half of

As none of the two possibilities can be ruled out using the information available in this statement and the question stem, the question cannot be answered using this statement alone. Statement (1) ALONE is NOT sufficient. Statement (2): Jim Bob purchased his first tricycle in , exactly two years before Billy Bob purchased his own tricycle. You can now calculate that exactly two years after would be , no matter what date or which month of the year Jim Bob purchased his first tricycle in. Consequently, you know the answer to the question using information available in this statement alone. Statement (2) ALONE is sufficient. The correct answer is B.

Rewrite

Question 70

2.893417

70. There were six members of John Joe Camaraderie Committee. Each of these six members was given at least one stamp. How many stamps was the youngest member of John Joe Camaraderie Committee given if stamps were given to these six members?

1. The youngest member and the oldest member of the Committee received the same number of stamps, which was less than the number of stamps received by each of the remaining four members. 2. The oldest member received one stamp, the second and third oldest members received two stamps each, and the fourth and fifth oldest members received equal numbers of stamps as each other.

(A) (B) (C) (D) (E)

Statement (1) ALONE is sufficient, but statement (2) alone is not sufficient. Statement (2) ALONE is sufficient, but statement (1) alone is not sufficient. BOTH statements TOGETHER are sufficient, but NEITHER statement ALONE is sufficient. EACH statement ALONE is sufficient. Statements (1) and (2) TOGETHER are NOT sufficient.

Question ID:

Word Problems
Solution This problem deals with integers, so you can try various alternate solutions out to check which one satisfies the requirements of the question. Six members received a total of stamps. You need to find out how many stamps the youngest member received. Statement (1): Read this statement carefully. According to this statement, the youngest member and the oldest member received the same number of stamps, and the remaining four members each received a high number of stamps. Since each member received at least one stamp and since each member only possess entire stamps, you know that the least number of stamps that the youngest and the oldest member each could receive is one. In this case, the four remaining members would receive the remaining ten stamps. However, if the youngest and the oldest members were to be given two stamps each, then there would remain only eight stamps to be given to the remaining four members. In that case, the

youngest and the oldest member would NOT receive fewer stamps than each of the four remaining members. This eliminates all possible values for the number of stamps received by the youngest and the oldest members except one. In other words, both the youngest member and the oldest member received one stamp each. This answers the question. Statement (1) ALONE is sufficient. Statement (2): According to this statement, the oldest member received one stamp, the next two oldest members received two stamps each and the next two oldest members received equal numbers of stamps as each other. The three oldest members received five stamps in total, leaving seven stamps for the three youngest members. If the fourth and fifth oldest members received two stamps each, then that would leave three stamps for the youngest member. On the other hand, if the fourth and fifth oldest members received three stamps each, for example, then that would leave just one stamp for the youngest member. Unless you know exactly how many stamps the fourth and the fifth oldest members each received, you can not answer the question. Statement (2) ALONE is NOT sufficient. The correct answer is A.

Rewrite

Question 71 71. If (1) (2)

-2.094527 , what does equal?

(A) (B) (C) (D) (E)

Statement (1) ALONE is sufficient, but statement (2) alone is not sufficient. Statement (2) ALONE is sufficient, but statement (1) alone is not sufficient. BOTH statements TOGETHER are sufficient, but NEITHER statement ALONE is sufficient. EACH statement ALONE is sufficient. Statements (1) and (2) TOGETHER are NOT sufficient.

Question ID:

Substitution
Solution Statement (1): The question stem tells you that and this statement tells you that . These are two equations in three unknowns which can not be solved to obtain the value of . More information is required to answer the question. You would need at least as many equations as unknowns unless the remaining unknowns except cancel out after some algebraic manipulation. In this case, at least another equation in these three unknowns is required. Statement (1) ALONE is NOT sufficient. Statement (2): This statement tells you that but no information is given on . Using this statement, you can determine the value of in this statement.

Statement (2) ALONE is NOT sufficient. BOTH statements TOGETHER: From statement , you can determine the value of . Plugging in this value of into equation would yield a linear equation in a single unknown, , which can be solved for . Once you find the value of , you can plug that value into equation and obtain a single linear equation in the unknown and consequently, solve for . BOTH statements TOGETHER are sufficient, but NEITHER statement ALONE is sufficient.. The correct answer is C.

Rewrite Question 72 72. What does (1) (2) 1.172467 equal?

(A) (B) (C) (D) (E)

Statement (1) ALONE is sufficient, but statement (2) alone is not sufficient. Statement (2) ALONE is sufficient, but statement (1) alone is not sufficient. BOTH statements TOGETHER are sufficient, but NEITHER statement ALONE is sufficient. EACH statement ALONE is sufficient. Statements (1) and (2) TOGETHER are NOT sufficient.

Question ID:

Roots of quadratic equations; Substitution


Solution You are asked to find the value of . In order to do so, you either need to find an expression containing or some variation of it, or you need two independent equations in these two unknowns. Statement (1): This statement tells you that . As equation provides you with no information on the

unknown , this statement is clearly not sufficient to answer the question. Statement (1) ALONE is NOT sufficient. Statement (2): In this statement, you are told that manipulating equation to obtain . There is no possible way of algebraically . This statement is also not sufficient.

Statement (2) ALONE is NOT sufficient.

BOTH statements TOGETHER: Plugging in from equation into equation , you obtain the equation: . Essentially, this tells you that must equal either or . Similarly, equation tells you that must equal or . However, as no unique pair of values of and can be determined, you can not find the exact value of using information in the two statements together. For example, if and , for instance, then this question. , then . On the other hand, if and . As a result, more information is required to answer

BOTH statements TOGETHER are NOT sufficient. The correct answer is E.

Rewrite Question 73 2.145798

73. Bob Brady and Jim Joe spent two nights at Hillbilly Hick s double-sized trailer bunk . The amounts due from Bob Brady and Jim Joe for two nights accommodation was and , respectively, where and are unknown digits from to , inclusive. The invoice was stained with grease as Hillbilly Hick handled the bill using his grease-stained hand. This caused the two digits and , respectively, in the invoice to be illegible (unreadable). What does equal?

1. 2.

(A) (B) (C) (D) (E)

Statement (1) ALONE is sufficient, but statement (2) alone is not sufficient. Statement (2) ALONE is sufficient, but statement (1) alone is not sufficient. BOTH statements TOGETHER are sufficient, but NEITHER statement ALONE is sufficient. EACH statement ALONE is sufficient. Statements (1) and (2) TOGETHER are NOT sufficient.

Question ID:

Digits
Solution

You are given two numbers which have two-digits each. The numbers are and , where and are unknown digits from to , inclusive. You are asked to find the value of . This can be done if you can find the values of and individually, or if you can find an expression which can be algebraically manipulated to equal . Statement (1): In this statement, the digits are reversed! You are given that . You should notice that for the sum of these two integers to equal , given that one of the numbers has a last digit of , the unknown digit must equal . There is no other way in which the two numbers could add upto . If equals , then the first number must equal and then you can easily calculate that the second number must equal . Thus, must equal and must equal . Finding is now straightforward: . Statement (1) ALONE is sufficient. Statement (2): According to this statement, . This inequality does not, in any way, help you determine the exact value of . For example, if and , then the inequality would read . On the other hand, if and , for instance, then the inequality would read . Both of these inequalities are feasible under the given restrictions and hence, no definitive answer to the question can be given using the information in this statement alone. Statement (2) ALONE is NOT sufficient. The correct answer is A.

Rewrite

Question 74

-2.122467

74. What is the radius of the circle shown in the above figure if the circle shown is inscribed in the square? (1) The area of the circle shown in the figure is (2) The area of the square shown in the figure is (A) (B) (C) (D) (E) Statement (1) ALONE is sufficient, but statement (2) alone is not sufficient. Statement (2) ALONE is sufficient, but statement (1) alone is not sufficient. BOTH statements TOGETHER are sufficient, but NEITHER statement ALONE is sufficient. EACH statement ALONE is sufficient. Statements (1) and (2) TOGETHER are NOT sufficient.

Question ID:

Square; Circumference; Area;


Solution You are asked to find the radius of the circle. This can be achieved if you know the diameter, or the circumference, or the area of the circle. Also, if you know the area of the square, then you could find the length of each side of the square and as the diameter of the inscribed circle is the same as the length of any side of the square, you could also calculate the diameter using that value. Statement (1): This statement provides you with the area of the inscribed circle. Consequently, you can find the radius of the circle by taking hint from the formula:   . Statement (1) ALONE is sufficient. Statement (2):

In this statement, you are provided the area of the square in which the circle is inscribed. As a result, you can also calculate the length of any side of this square, and that would be the same as the diameter of the circle. Dividing the diameter of the circle by would produce the radius of the circle. Statement (2) ALONE is sufficient. EACH statement ALONE is sufficient. The correct answer is D.

Rewrite Question 75 75. What does equal? 0.025397

1. 2. (A) (B) (C) (D) (E) Statement (1) ALONE is sufficient, but statement (2) alone is not sufficient. Statement (2) ALONE is sufficient, but statement (1) alone is not sufficient. BOTH statements TOGETHER are sufficient, but NEITHER statement ALONE is sufficient. EACH statement ALONE is sufficient. Statements (1) and (2) TOGETHER are NOT sufficient.

Question ID:

Value of an expression
Solution The question can not be simplified further: What does equal? As no value for is provided in the question stem, you must look into the statements for more information. Statement (1): In this statement, you are given that . This is a linear equation in two unknowns ,

which can not be solved for unless more information, such as the value of or the value of is provided. Statement (1) ALONE is NOT sufficient.

Statement (2): According to this statement, . This equation may look like

a complicated, quadratic equation in two unknowns which can not be solved for . However, if you notice carefully, you would observe that equation may be rewritten as: from both sides of this equation yields: . This answers the question. Statement (2) ALONE is sufficient. The correct answer is B. , when expanded, equals . Thus, . Subtracting , which can be solved to obtain

Question 76

-1.042387

76. If each of

and

are distinct, nonzero constants, is

1. 2.


(A) (B) (C) (D) (E)

Statement (1) ALONE is sufficient, but statement (2) alone is not sufficient. Statement (2) ALONE is sufficient, but statement (1) alone is not sufficient. BOTH statements TOGETHER are sufficient, but NEITHER statement ALONE is sufficient. EACH statement ALONE is sufficient. Statements (1) and (2) TOGETHER are NOT sufficient.

Question ID:

Linear inequalities
Solution You are asked to determine if . This may look like a complicated expression

but closer inspection reveals that this expression can be simplified greatly. Carrying out simple . algebraic division, you would obtain the equivalent inequality: Hence, the question merely asks you if the product of these three expressions is negative. As and must both be positive since none of and are zero and none of them are equal to each other, essentially you need to find out if . Adding to both sides of the last inequality, you obtain the equivalent inequality: . Your task becomes much simpler now. You only need to find out if Statement (1): This statement tells you that and tells you nothing about the relationship between and . .

Statement (1) ALONE is NOT sufficient. Statement (2): In this statement, you are told that question essentially asked you if Statement (2) ALONE is sufficient. The correct answer is B. . This answers the question in the negative since the as you found out in the solution worked out above.

Rewrite Question 77 -2.139867 ?

77. What is the slope of the line passing through the point

1. Line 2. Line

is parallel to line , which has a slope of intersects the at the point

(A) (B) (C) (D) (E)

Statement (1) ALONE is sufficient, but statement (2) alone is not sufficient. Statement (2) ALONE is sufficient, but statement (1) alone is not sufficient. BOTH statements TOGETHER are sufficient, but NEITHER statement ALONE is sufficient. EACH statement ALONE is sufficient. Statements (1) and (2) TOGETHER are NOT sufficient.

Question ID:

Slope
Solution The slope of any line in the rectangular coordinate axes could be determined in many different ways. For example, if any two points on line could be determined, then the slope could be determined by finding the
    

from the first point to the second. Also, the slope

of any line parallel to line is the same as the slope of line . There are many possible ways of determining the slope of any line in the rectangular coordinate axes. Statement (1): Since line has a slope of . and is parallel to line , you can assert that line also has a slope of

Statement (1) ALONE is sufficient. Statement (2): The question stem provides you with one point on line and this statement provides you with another point. Together, the two points provide you with sufficient information to determine the slope of line . Statement (2) ALONE is sufficient.

EACH statement ALONE is sufficient. The correct answer is D.

Rewrite Question 78 -1.092568

78. Country and country are two of the largest developing countries in the world. If the two countries have a total number of billion poor people, then what is the poor population of country ?

1. Country has 2. Country

billion more poor people than country  percent of the global poor population

accounts for

(A) (B) (C) (D) (E)

Statement (1) ALONE is sufficient, but statement (2) alone is not sufficient. Statement (2) ALONE is sufficient, but statement (1) alone is not sufficient. BOTH statements TOGETHER are sufficient, but NEITHER statement ALONE is sufficient. EACH statement ALONE is sufficient. Statements (1) and (2) TOGETHER are NOT sufficient.

Question ID:

System of independent, linear equations


Solution Let the number of poor people in countries question stem informs you that Statement (1): In this statement, you are informed that . Together with equation , equation forms a system of two linearly independent equations which can be solved to obtain the values of and . Hence, this statement provides you with sufficient information. Statement (1) ALONE is sufficient. Statement (2): This statement tells you that is percent of the global poor population. If you let the number of poor people in the world equal , then this statement can be algebraically represented as: and be billion and billion, respectively. The . You are asked to determine the value of .

. Together with equation

, equation

forms a system of two equations in three

unknowns which can not be solved for without additional information. If you were provided the value of or , then you could find the value of using equations and . Statement (2) ALONE is NOT sufficient. The correct answer is A. Additional Notes A system of linear equations is said to be independent if none of the equations can be algebraically manipulated to look like the other. In essence, this means none of the equations repeats information already provided by the other equations.

Rewrite Question 79 0.076254

r r r

r r r r

79. What does equal in the figure above?

1. 2.

(A) (B) (C) (D) (E)

Statement (1) ALONE is sufficient, but statement (2) alone is not sufficient. Statement (2) ALONE is sufficient, but statement (1) alone is not sufficient. BOTH statements TOGETHER are sufficient, but NEITHER statement ALONE is sufficient. EACH statement ALONE is sufficient. Statements (1) and (2) TOGETHER are NOT sufficient.

Question ID:

Parallel; Intersecting; Vertical, Adjacent; Alternate angles; Corresponding


Solution Whenever a pair of parallel lines is intersected by a transversal (non-parallel line), the acute angles formed are all equal to each other and the obtuse angles formed are also equal to each other. If the transversal (non-parallel line) is perpendicular to the pair of parallel lines, then all the eight angles formed are right-angles. Hence, you know that Statement (1): Here, you are told that . Subtracting Alternatively, . As , as well and since . , , you know that , and .

from both sides of this equation, you get:

since they form a straight line. Thus, you may write that

from which calculating the value of is straightforward. Statement (1) ALONE is sufficient. Statement (2): According to this statement, . As these two angles form a straight line, you may also write that . Together, these two equations form a pair of independent, linear equations which can be solved for . Once you find the value of , finding is straightforward since . Statement (2) ALONE is sufficient. EACH statement ALONE is sufficient. The correct answer is D.

Rewrite Question 80 80. What does 0.063524 equal?

1. 2.


(A) (B) (C) (D) (E)

Statement (1) ALONE is sufficient, but statement (2) alone is not sufficient. Statement (2) ALONE is sufficient, but statement (1) alone is not sufficient. BOTH statements TOGETHER are sufficient, but NEITHER statement ALONE is sufficient. EACH statement ALONE is sufficient. Statements (1) and (2) TOGETHER are NOT sufficient.

Question ID:

Value of an expression; Formulas of short multiplication


Solution The question asks you to find the value of could rewrite this expression as Statement (1): According to this statement, . The last equation is simply essence, this tells you that As . would also equal . Hence, . Subtracting , or from both sides yields: . In . Using the Difference of Squares Formula, you .

, any expression multiplied by as well. This answers the question.

Statement (1) ALONE is sufficient. Statement (2): According to this statement, , however, you are not provided the value of either , or , or , or . Thus, you can not evaluate using the information in this statement alone. Statement (2) ALONE is NOT sufficient. The correct answer is A.

Rewrite Question 81 1.163597

81. What is the area of the shaded region in the figure shown above if circle? (1) , and shown above is

is the centre of the

(2) The area of the circle with centre (A) (B) (C) (D) (E)

Statement (1) ALONE is sufficient, but statement (2) alone is not sufficient. Statement (2) ALONE is sufficient, but statement (1) alone is not sufficient. BOTH statements TOGETHER are sufficient, but NEITHER statement ALONE is sufficient. EACH statement ALONE is sufficient. Statements (1) and (2) TOGETHER are NOT sufficient.

Question ID:

Right triangle, Pythagorean Theorem, special right triangles ; Sector


Solution The shaded region in the figure shown below does not form a known regular geometrical shape. However, if you consider the area of this region as the area of the entire circle less the area of the region within the circle that is covered by the triangle, then you only need to know two things: the area of the circle itself, and the angle subtended by at the centre of the circle. Once you know the angle subtended (created) by at the centre of the circle, you can calculate the area of the shaded region accordingly.

Statement (1): This statement tells you that , and . This is a typical Pythagorean right-triangle you may be familiar with. If not, you can still confirm that confirming that the sides of triangle are that of a right-triangle, according to Pythagorean Theorem. Therefore, you know that angle is a right-angle. However, you still do not know the area of the circle with centre . Without knowing the area of the circle, you can not calculate the area of the shaded region. Statement (1) ALONE is NOT sufficient. Statement (2): This statement informs you that the area of the circle is . However, this does not provide any information on the angle subtended (created) by at the centre of the circle. Statement (2) ALONE is NOT sufficient. BOTH statements TOGETHER: From statement , you know that the angle subtended (created) by at the centre of the circle is a right-angle and that the area of the circle is . Consequently, you can calculate the area of the shaded region.
   

BOTH statements TOGETHER are sufficient but NEITHER statement ALONE is sufficient. The correct answer is C.

Rewrite

Question 82 82. Is (1) (2) (A) (B) (C) (D) (E) ?

-1.213154

Statement (1) ALONE is sufficient, but statement (2) alone is not sufficient. Statement (2) ALONE is sufficient, but statement (1) alone is not sufficient. BOTH statements TOGETHER are sufficient, but NEITHER statement ALONE is sufficient. EACH statement ALONE is sufficient. Statements (1) and (2) TOGETHER are NOT sufficient.

Question ID:

Linear Inequalities
Solution The question is easy to understand: Is Statement (1): According to this statement, . Subtracting from both sides of this inequality yields: but not whether . For example, ?

. Hence, this statement tells you that if , then but and as well.

. On the other hand, if

, for instance, then

As a result, this statement does not tell you definitively whether required to answer this question. Statement (1) ALONE is NOT sufficient. Statement (2): In this statement, you are told that

. More information is

. Adding to both sides yields the equivalent inequality: . As a result, you can not answer , then but , for

. This statement essentially informs you that

the question using the information in this statement alone. For example, if

. Hence, the answer to the question would be negative. On the other hand, if instance, then and .

Thus, this information does not provide you enough information to answer the question. Statement (2) ALONE is NOT sufficient. BOTH statements TOGETHER: The two statements together tell you that determine if example, then information. BOTH statements TOGETHER are NOT sufficient. The correct answer is E. . For instance, if and , then . However, this is still not sufficient to . On the other hand, if , for

. Therefore, you still can not determine if

without additional

Rewrite Question 83 2.163957

83. What does equal if (1) (2) (A) (B) (C) (D) (E) Statement (1) ALONE is sufficient, but statement (2) alone is not sufficient. Statement (2) ALONE is sufficient, but statement (1) alone is not sufficient. BOTH statements TOGETHER are sufficient, but NEITHER statement ALONE is sufficient. EACH statement ALONE is sufficient. Statements (1) and (2) TOGETHER are NOT sufficient.

Question ID:

Formulas of short multiplication; Substitution


Solution You are given that . Expanding the right side of this equation, you get: . Equating the coefficients of this expression with the left side of the given equation, you can infer that and that .

The question asks you to determine the value of . You can determine that if you know the value of or if you are given the value of . However, caution must be taken since could be either positive or negative. Statement (1): This statement tells you that . From the calculations above, you know that . Hence, you know that . Subtracting from both sides of this equation produces the equation: . Factoring out from the left side of this expression turns the equation into: . Hence, either or . In other words, could equal or . Thus, could equal or . More information is required to answer the question.

Statement (1) ALONE is NOT sufficient. Statement (2): According to this statement, . From the calculations in the solution above, you know that . Hence, you know that or . As a result, could equal or could equal . More information is required to determine the value of . Statement (2) ALONE is NOT sufficient. BOTH statements TOGETHER: From statement , you obtained that could equal or , and from statement , you obtained that could equal or . Combining the information in these two statements together, you observe that could only equal since that is the only solution common from these two statements. Hence, must equal . This answers the question. BOTH statements TOGETHER are sufficient, but NEITHER statement ALONE is sufficient. The correct answer is C.

Rewrite Question 84 84. Is ? 1.093754

(1) (2) (A) (B) (C) (D) (E)

added to two times decreased by equals

equals

Statement (1) ALONE is sufficient, but statement (2) alone is not sufficient. Statement (2) ALONE is sufficient, but statement (1) alone is not sufficient. BOTH statements TOGETHER are sufficient, but NEITHER statement ALONE is sufficient. EACH statement ALONE is sufficient. Statements (1) and (2) TOGETHER are NOT sufficient.

Question ID:

Word Problems; Linear Inequalities


Solution Statement (1): According to this statement, added to two times equals . Algebraically, this can be written as: . Subtracting from both sides, you obtain: . This directly answers the question. Since , . Statement (1) ALONE is sufficient. Statement (2): This statement actually tells you the same thing as the first statement. In this statement, you are told that decreased by equals . Algebraically, this tells you that . Multiplying . As you from both sides yields: this equation by throughout and then adding to both sides yields: can see, this is the same equation as equation and subtracting . Hence, . Statement (2) ALONE is sufficient. EACH statement ALONE is sufficient. The correct answer is D.

Rewrite Question 85 85. Does -1.043626 equal ?

(1) (2) (A) (B) (C) (D) (E) Statement (1) ALONE is sufficient, but statement (2) alone is not sufficient. Statement (2) ALONE is sufficient, but statement (1) alone is not sufficient. BOTH statements TOGETHER are sufficient, but NEITHER statement ALONE is sufficient. EACH statement ALONE is sufficient. Statements (1) and (2) TOGETHER are NOT sufficient.

Question ID:

Value of an expression
Solution The question asks for the value of could evaluate . Thus, if you can find the values of all three unknowns, you and , or

. You could also do the same if you found the values of and , or

and , for examples. Statement (1): According to this statement, . There is no mention of in this statement. Therefore, it is clearly

not sufficient to answer the question. Statement (1) ALONE is NOT sufficient. Statement (2): According to this statement, . There is no mention of in this statement. Therefore, it is clearly

not sufficient to answer the question. Statement (2) ALONE is NOT sufficient. BOTH statements TOGETHER: From the first statement, you know that equals and from the second statement, you know that equals However, there is still no way to know the value of . On the other hand, if
and

. If

and

, for . As

example, then

, for instance, then

there is more than one possible answer to the question based on the information provided, more information is required to answer the question correctly. BOTH statements TOGETHER are NOT sufficient. The correct answer is E.

Rewrite Question 86 0.034198

86. What does equal? (1) (2) (A) (B) (C) (D) (E) Statement (1) ALONE is sufficient, but statement (2) alone is not sufficient. Statement (2) ALONE is sufficient, but statement (1) alone is not sufficient. BOTH statements TOGETHER are sufficient, but NEITHER statement ALONE is sufficient. EACH statement ALONE is sufficient. Statements (1) and (2) TOGETHER are NOT sufficient.

Question ID:

Roots of quadratic equations; Additional restrictions, typical tricks


Solution You need to find the value of . No information regarding the value of stem, so let us look at the statements for more information. Statement (1): You are given that . This is a cubic (third-order) equation in a single unknown, which can have upto three distinct roots or zeroes . Unless there is exactly one zero or root of this equation, you can not determine the value of . Subtracting from both sides of equation produces the equation: . Factoring out from the left side of this equation yields: . Using reverse FOIL to factorize , you obtain: . Thus, the last equation turns into: . Hence, either or or . As a result, either , or or . There are three distinct values of that appear in the solution based on the information in this statement. More information is required. Statement (1) ALONE is NOT sufficient. Statement (2): is given in the question

According to this statement, . This is a cubic, third-degree equation in a single unknown which can have to upto three distinct roots or zeroes . Unless there is exactly one root or zero of this equation, you can not determine the value of . Subtracting produces: or from both sides of equation yields: . Using reverse FOIL to factorize . Thus, the last equation transforms into: or . As a result, either or or . Factoring out , you obtain: . Hence, either .

Since more than one value of appears in the solution of the equation given in this statement, you can not determine a unique value of using the information in this statement alone. Statement (2) ALONE is NOT sufficient. BOTH statements TOGETHER: From statement , you deduced that could equal any of or . From statement , you deduced that could equal any of or . The only common value of that you could deduce from both of these statements is . In other words, both the equations are satisfied only if . Thus, must equal . BOTH statements TOGETHER are sufficient but NEITHER statement ALONE is sufficient. The correct answer is C.

Das könnte Ihnen auch gefallen